What is Eigenstates: Definition and 192 Discussions

In quantum physics, a quantum state is a mathematical entity that provides a probability distribution for the outcomes of each possible measurement on a system. Knowledge of the quantum state together with the rules for the system's evolution in time exhausts all that can be predicted about the system's behavior. A mixture of quantum states is again a quantum state. Quantum states that cannot be written as a mixture of other states are called pure quantum states, while all other states are called mixed quantum states. A pure quantum state can be represented by a ray in a Hilbert space over the complex numbers, while mixed states are represented by density matrices, which are positive semidefinite operators that act on Hilbert spaces.Pure states are also known as state vectors or wave functions, the latter term applying particularly when they are represented as functions of position or momentum. For example, when dealing with the energy spectrum of the electron in a hydrogen atom, the relevant state vectors are identified by the principal quantum number n, the angular momentum quantum number l, the magnetic quantum number m, and the spin z-component sz. For another example, if the spin of an electron is measured in any direction, e.g. with a Stern–Gerlach experiment, there are two possible results: up or down. The Hilbert space for the electron's spin is therefore two-dimensional, constituting a qubit. A pure state here is represented by a two-dimensional complex vector



(
α
,
β
)


{\displaystyle (\alpha ,\beta )}
, with a length of one; that is, with





|

α


|


2


+

|

β


|


2


=
1
,


{\displaystyle |\alpha |^{2}+|\beta |^{2}=1,}
where




|

α

|



{\displaystyle |\alpha |}
and




|

β

|



{\displaystyle |\beta |}
are the absolute values of



α


{\displaystyle \alpha }
and



β


{\displaystyle \beta }
. A mixed state, in this case, has the structure of a



2
×
2


{\displaystyle 2\times 2}
matrix that is Hermitian and positive semi-definite, and has trace 1. A more complicated case is given (in bra–ket notation) by the singlet state, which exemplifies quantum entanglement:





|
ψ


=


1

2





(



|

↑↓





|

↓↑





)


,


{\displaystyle \left|\psi \right\rangle ={\frac {1}{\sqrt {2}}}{\big (}\left|\uparrow \downarrow \right\rangle -\left|\downarrow \uparrow \right\rangle {\big )},}
which involves superposition of joint spin states for two particles with spin 1⁄2. The singlet state satisfies the property that if the particles' spins are measured along the same direction then either the spin of the first particle is observed up and the spin of the second particle is observed down, or the first one is observed down and the second one is observed up, both possibilities occurring with equal probability.
A mixed quantum state corresponds to a probabilistic mixture of pure states; however, different distributions of pure states can generate equivalent (i.e., physically indistinguishable) mixed states. The Schrödinger–HJW theorem classifies the multitude of ways to write a given mixed state as a convex combination of pure states. Before a particular measurement is performed on a quantum system, the theory gives only a probability distribution for the outcome, and the form that this distribution takes is completely determined by the quantum state and the linear operators describing the measurement. Probability distributions for different measurements exhibit tradeoffs exemplified by the uncertainty principle: a state that implies a narrow spread of possible outcomes for one experiment necessarily implies a wide spread of possible outcomes for another.

View More On Wikipedia.org
  1. B

    Finding eigenstates and eigenvalues of hamiltonian

    Hey there, the question I'm working on is written below:- Let |a'> and |a''> be eigenstates of a Hermitian operator A with eigenvalues a' and a'' respectively. (a'≠a'') The Hamiltonian operator is given by: H = |a'>∂<a''| + |a''>∂<a'| where ∂ is just a real number. Write down the eigenstates...
  2. O

    What are the eigenstates of the anti-ferromagnetic dimer Hamiltonian?

    Homework Statement The hamiltonian of a simple anti-ferromagnetic dimer is given by H=JS(1)\bulletS(2)-μB(Sz(1)+Sz(2)) find the eigenvalues and eigenvectors of H. Homework Equations The Attempt at a Solution The professor gave the hint that the eigenstates are of...
  3. A

    Eigenstates of the momentum operator

    For the free particle the solution to the SE are eigenstates of the momentum. You get something like: ψ = Aexp(ik(x-vt)) + Bexp(-ik(x+vt)) , where k is a constant And my book then says that first term represents a wave traveling to the right and the second a wave traveling to the left. But I...
  4. M

    Energy vs. position eigenstates

    This might be a silly question, but I'm not sure about the answer... Commuting operators have identical eigenstates. For example: energy and position operators seem to commute: [E,x] = Ex - xE = 0 Does it mean that position and energy operators share identical eigenstates? Because...
  5. A

    Particle in superposition of energy eigenstates and conservation of energy.

    When a particle is in superposition of energy eigenstates and has a probability of being found in either state, what does that say about the energy of the particle and conservation of energy. What I mean is, since the energy eigenstates have different energy values, where's the rest of the...
  6. K

    Non-strange non-baryonic states are eigenstates of G-parity

    It is said that all non-strange non-baryonic states are eigenstates of G-parity. And all members of an isospin multiplet have the same eigenvalue. Can anyone give me a proof to these two statements, or show me where I can find one? In addition, the composite state consisting of K^{+}K^{-} should...
  7. I

    Using the rotation operator to solve for eigenstates upon a general basis

    Homework Statement I need to express the rotation operator as follows R(uj) = cos(u/2) + 2i(\hbar) S_y sin(u/2) given the fact that R(uj)= e^(iuS_y/(\hbar)) using |+-z> as a basis, expanding R in a taylor series express S_y^2 as a matrix Homework Equations I know...
  8. C

    Superposing Energy Eigenstates.

    Homework Statement Im trying to understand what happened in this book they have |I>=\frac{|1>}{\sqrt{2}}+\frac{|2>}{\sqrt{2}} and then |II>=\frac{|1>}{\sqrt{2}}-\frac{|2>}{\sqrt{2}} and then they say they superimpose these equations to get...
  9. S

    Quantum Physics: observables, eigenstates and probability

    Homework Statement Observable \widehat{A} has eigenvalues \pm1 with corresponding eigenfunctions u_{+} and u_{-}. Observable \widehat{B} has eigenvalues \pm1 with corresponding eigenfunctions v_{+} and v_{-}. The eigenfunctions are related by: v_{+} = (u_{+} + u_{-})/\sqrt{2} v_{-} =...
  10. P

    Energy Eigenstates of a Perturbed Quantum Harmonic Oscillator

    Homework Statement (See attachment) Homework Equations x = \sqrt{\frac{\hbar}{2m \omega}} ( a + a^{\dagger} ) x = i \sqrt{\frac{\hbar m \omega}{2}} ( a^{\dagger} - a ) The Attempt at a Solution In part a) I was able to construct a separable Hamiltonian for the harmonic...
  11. W

    Uncertainty in position for eigenstates (Liboff 3.10)

    Homework Statement From Liboff edition 4: For the state ψ(x,t)=A exp(x-x0)2 / {4a2} * exp(ip0x)/hbar * exp (-iω0t) show that (Δx)2 = a2 then argue the consistency of this conclusion with the change in shape that |ψ2| suffers with a change in the parameter a Homework...
  12. N

    A question about mass eigenstates

    I am very confuse about mass eigenstate. In some books they say'' If neutrinos are massless then lepton mixing is unobservable.Any Cabibbo-like rotation still leaves us with neutrino mass eigenstates''. I do not understand that statement.Why the mixing of some states gives a state being mass...
  13. J

    Eigenstates of Dirac Potential in Momentum Space

    Homework Statement Consider a particle moving in one dimension and bound to an attractive Dirac δ-function potential located at the origin. Work in units such that m=\hbar=1. The Hamiltonian is given, in real (x) space, by: H=-\frac{1}{2}\frac{d^2}{dx^2}-\delta (x) The (non normalized)...
  14. L

    What are the eigenstates of quantum fields?

    I believe that in qft the particle states are eigenvectors of the Hamiltonian(which also commutes with the number operator), i.e. H|n> = E|n>. A Fock vector is a product of many particle state vectors. But what are the eigenvectors of psi(x)? thanks
  15. M

    Why aren't u, c, and t quarks a mixture of mass eigenstates?

    Weak eigenstates d', s', b' are a mixture of mass eigenstates. For example, s'=V_cd*d+V_cs*s+V_cb*b This doesn't seem to be the case for u, c, t quarks. For example, there is no c'=V_su*u+V_sc*c+V_st*t Why is that?
  16. P

    Map energy eigenstates to cartesian unit vectors - Harmonic Osillator

    Homework Statement Evaluate the matrix elements x_{nn'} = \left<n\left|x\right|n'\right> and p_{nn'} = \left<n\left|p\right|n'\right> and map the energy eigenstates \left|n\right> to Cartesian unit vectors. Homework Equations x = \sqrt{\frac{\hbar}{2m...
  17. L

    Can an operator without complete eigenstates be measured?

    Say you have some operator A with an incomplete set of eigenstates, but the state of the system is such that it happens to be expressible as a sum (possibly infinite, or integral, whatever) of the eigenstates of A, and let's say the eigenvalues are real and whatever is necessary...we may assume...
  18. H

    Can changes in spring constant affect the eigenstates of a harmonic oscillator?

    We now that if [A,B]=0, they have the same eigenstates. But consider a harmonic oscillator with the spring constant k1. If we change k1 to k2, then [H1,H2]=0 and the above expression implies that the eigenstates should not change while they really change! Could you please tell me if i am wrong?
  19. T

    The difference between the weak and mass eigenstates in the PNMS matrix

    Hi, I am hoping someone could clear up a few things about neutrinos oscillations for me. For the sake of this dicussion let's set up the neutrino mixing equations in such a way that the flavor eigenstates are a superposition of mass eigenstates. So now for example we have...
  20. N

    Why are nuclear excited states always eigenstates of parity?

    QM says that states which are simultaneous eigenstates of two commuting observables are allowed. If you don't have such states to start with you can construct them with the Gramm-Schmidt orthogonalization procedure. Consider the excited states of a nucleus. (They can be considered eigenstates...
  21. Q

    Functions with operator valued arguments acting on eigenstates

    This question concerns the outcome when operator valued functions act on an energy eigenstate. Given an eigenstate at t =0, say |Ej > , I have seen or inferred in some of the literature that the following applies : exp(-iHt/h) |Ej > = exp(- iEj t/h) |Ej > Where h = h-bar Ej is energy...
  22. J

    What is the connection between energy eigenstates and position?

    The first thing I remember hearing about in QM was the time-independent 1-D schrodinger equation, Hψ = (\frac{-\hbar^2}{2m}\frac{d^2}{dx^2} + V)ψ(x) = Eψ(x) . This is an eigenvalue equation, the Hamiltonian operator H operating on the energy eigenstate ψ to produce the product of the energy...
  23. jfy4

    What is the result of J_x, J_y, and [J_x,J_y] acting on a specific eigenstate?

    Homework Statement Let |0,0\rangle be the simultaneous eigenstate of \mathbf{J}^2 and J_z with eigenvalues 0 and 0. Find J_x|0,0\rangle \quad\quad J_y |0,0\rangle \quad\quad [J_x,J_y]|0,0\rangle 2. The attempt at a solution It seemed reasonable to write J_x and J_y in terms of ladder...
  24. T

    Eigenvalues and Eigenstates of Spin Operator

    I'm not exactly looking for help finding the eigenvalues of the spin operator, I'm mainly wondering if there is a better technique to do it. Homework Statement Find the eigenvalues and corresponding eigenstates of a spin 1/2 particle in an arbitrary direction (θ,\phi) using the Pauli...
  25. C

    Eigenstates of composite spin hamiltoninan

    A spin-3/2 particle (labeled 1) and a spin-1/2 particle (labeled 2) are interacting via the Hamiltonian \lambda \vec S_1 \cdot \vec S_2 + g(S_{1z} + S_{2z})^2 where S1 is the spin operator of particle 1 (the spin-3/2) and S2 is the spin operator of particle 2 (the spin-1/2). λ and g...
  26. R

    Energy Eigenstates: Can Electron in Hydrogen Atom?

    Can an electron in a hydrogen atom every been in an energy eigenstate?
  27. T

    Free particle - Eigenstates expansion

    Hi everybody! Two question for you: 1) Take a free particle, moving in the x direction. Its (time indipendent) wave function, in terms of the momentum is \psi(x)=\frac{e^{i\frac{p}{\hbar}x}}{\sqrt{2\pi\hbar}}. Now, i know the momentum of the particle: p. So i should not know anything about its...
  28. D

    Understanding the Relationship Between Neutrino Flavours and Mass Eigenstates

    Could somebody please explain to me the difference between flavour and mass eigenstates. The question is "Neutrinos can be produced from charged pion decay, What force is involved, and at the time of creation is the neutrino in a flavour or mass eigenstate, and why?" And why would it be...
  29. G

    Continuous eigenstates vs discrete eigenstates

    "Continuous eigenstates" vs "discrete eigenstates" There's this thing that's bothering me: if I have an Hamiltonian with a discrete and continuous spectrum, every book I read on quantum mechanics says that eigenvectors of discrete eigenvalues are orthogonal in the "Kronecker sense" (their...
  30. J

    Exploring the Relationship between Energy Eigenstates and Wavefunctions

    Why do we associate the energy eigenstates with the "wavefunction" and "position"? This is something that has bothered me for a long time but that I never got around to asking. I suspect I'll feel like an idiot for not knowing this but anyway... One of the first things I was introduced to...
  31. C

    Quantum mechanics, way of writing the eigenstates

    Homework Statement I'm having some trouble understanding exactly how the eigenstates of this matrix are being presented. A= ( 0 -i i 0 ) <- matrix I can find the eigenvalues to be +/- 1 which gives the eigenvectors to be x=-iy or x=iy. The eigenvectors are then being...
  32. R

    Overlap integrals and eigenstates problem

    To find the probability of a particle being at position x we use <\Psi|\Psi> where the complex conjugate ensures that the answer is real. This means that we're looking at the square of the wave function to determine the probability of finding the particle. Now to determine the probability...
  33. C

    Solving Operator Eigenstates: Constants & Normalization

    Homework Statement I have some questions about the eigenstates of an operator. A state is an eigenstate of an operator if the application of the operator on the state results in a constant complex multiple of the state. The constant complex multiple will be the eigenvalue. For instance...
  34. K

    Representing Wavefunction as Superposition of Eigenstates

    Homework Statement A particle in the infinite square well with V(x)=0 for 0<x<a and V(x)=infinity otherwise has the initial (t=0) wave function: psi(x,0)=Ax for 0<x<a/2 psi(x,0)= A(a-x) for a/2<x<a 1) Sketch psi and psi^2 (DONE) 2) Determine A [DONE - 2*sqrt(3)*a^(-3/2)] 3) Find psi(x,t)...
  35. L

    QM- Probability of finding a system in a superposition of energy eigenstates.

    Homework Statement A system at time t = 0 is in the state |ψ> = a|E1> + b|E2>, where |E1> and |E2> are (normalised) energy eigenstates with two different energies E1 and E2, and a, b are real numbers. Write down the state |ψ, t> for the system at time t. What are the probabilities at time t to...
  36. F

    Prove that eigenstates of hermitian operator form a complete set

    Not really sure how to go about this. Our lecture said "it can be shown" but didn't go into any detail as apparently the proof is quite long. I'd really appreciate it if someone could show me how this is done. Thanks. (Not sure if this is relevant but I have not yet studied Hilbert spaces).
  37. Shackleford

    How are (u,v) and alpha+ related in the Eigenstates of Spin 1/2?

    How did they get (u,v) in the second line and then alpha+? http://i111.photobucket.com/albums/n149/camarolt4z28/2010-11-20113911.jpg?t=1290274889
  38. P

    Infinite Square Well Eigenstates

    Homework Statement The eigenstates of the infinite square well are not energy eigenstates and are not momentum eigenstates. Homework Equations The Attempt at a Solution I don't understand how this can be? If the eigenstates of the infinite square well are energy eigenstates...
  39. A

    Eigenstates in quantum mechanics

    Homework Statement 1. Is state \psi_{0,2,1}-\psi_{5,0,1} an eigenstate of L_{x} 2. Is state \psi_{1,3,1}-\psi_{4,2,0} an eigenstate of L^{2} Homework Equations Stationary state of Hamiltonian defined by: [itex]\psi_{n,l,m}[/tex] where the subscripts denote quantum numbers. The...
  40. JK423

    Question on Completeness of a set of Eigenstates

    I know that the set of eigenstates of an operator forms a complete set, a basis. Which means that any state can be written as a linear sum of the eigenstates. But is the word 'any' correct? I haven't seen a proof of the above completeness theorem, so i don't know what the mathematical...
  41. G

    Question concerning Hamiltonian and eigenstates

    Homework Statement Two spin-1/2 particles are placed in a system described by Hamiltonian H=S(x1)S(x2), (S(x) being the spin operator in the x-direction). States are written like |\uparrow\downarrow>, (and can be represented by 2 x 2 matrix) so that there are 4 possible states...
  42. M

    Interaction betwen position eigenstates of particle

    If a (charged) particle is in superposition of two different positions A and B is it interacting with itself by electric forces (and gravitational force)? Is it true that the particle is in both places A and B simultaneously?
  43. Y

    Quick noob question: commutative of eigenstates

    let L be angular momentum operator. [L^2 , Lz] = 0 [L^2 , Lx] = 0 (I haven't prove this, but appearantly it's correct according to lecturer) does it imply that [Lx , Lz] = 0? this is just one interesting thoughts that cross my mind because I recalled that if 2 matrix [A,B] =0, A and B...
  44. D

    Angular Momentum eigenstates, and tensor products

    This is taken from a text problem, but I am putting it in this section because I think my question goes beyond the problem itself: If a particle has a wave function psi = A*R(r)*cos2 (theta), for example, then if I want to find the probability that its angular momentum is l I would find the...
  45. J

    Identical particles, eigenstates, empirical or theoretical

    My question is that is the fact, that all particles are either boson or fermions, only an empirical fact, or can it be argued theoretically too. The reason why I'm asking this is that I have not encountered anyone stating honestly, that it is an empirical fact only. But on the other hand, I...
  46. N

    Orthonormal Energy Eigenstates

    I'm a self learner with a decent math background trying to follow this lecture. I have been able to follow him perfectly up to this point but at 16:00 Professor Balakrishnan states that "<n|a|n> = 0" because |n-1> is perpendicular to |n>. My question is why and how do we know that an...
  47. W

    Anyone solved eigenstates in the field of a magnetic monopole?

    any references? i think it is a basic problem
  48. Y

    Are Fock States Eigenstates for the Operators a^{\dagger} and a?

    Homework Statement I have to show that the Fock states are eigenstates for the operators {{\hat{a}}^{\dagger }} and/or {\hat{a}} And I'm not totally sure how to show this. Homework Equations ? The Attempt at a Solution I know that if I use the operators on a random Fock...
  49. M

    Perturbation: First order correction to particle-in-box eigenstates

    First order correction to particle-in-box eigenstates for Dirac perturbation Homework Statement Calculate the first three nonzero terms in the expansion of the correction to the ground state \psi^{1}_{1} for a Dirac delta perturbation of strength alpha at a/2 (box from 0 to a). Homework...
  50. D

    What is a useful way to talk about eigenstates of the position operator

    So I've been having a specific major hang-up when it comes to understanding basic quantum mechanics, which is the position operator. For the SHO, the time independent Schroedinger's equation looks like E\psi = \frac{\hat{p}^2}{2m}\psi + \frac{1}{2}mw^2\hat{x}^2\psi Except that...
Back
Top